Source: Of course

This topic has expert replies
User avatar
GMAT Instructor
Posts: 3650
Joined: Wed Jan 21, 2009 4:27 am
Location: India
Thanked: 267 times
Followed by:80 members
GMAT Score:760

Source: Of course

by sanju09 » Tue Jul 27, 2010 6:30 am
Is x - y > 0?
(1) 4 < 1/x < 10.
(2) -3 < 1/y < 6.


[spoiler]Source: Of course: writegmat.com[/spoiler]
The mind is everything. What you think you become. -Lord Buddha



Sanjeev K Saxena
Quantitative Instructor
The Princeton Review - Manya Abroad
Lucknow-226001

www.manyagroup.com

User avatar
Legendary Member
Posts: 1893
Joined: Sun May 30, 2010 11:48 pm
Thanked: 215 times
Followed by:7 members

by kvcpk » Tue Jul 27, 2010 6:35 am
sanju09 wrote:Is x - y > 0?
(1) 4 < 1/x < 10.
(2) -3 < 1/y < 6.


[spoiler]Source: Of course: writegmat.com[/spoiler]
1 is INSUFF.. not info about y
2 is INSUFF.. no info about x

Combining:
let x=1/5, y = 1/5.5
x>y
let x=1/9, y = 1/5
x<y
2 different answers

Hence INSUFF

pick E

Newbie | Next Rank: 10 Posts
Posts: 1
Joined: Tue Jul 27, 2010 8:37 am

by Sidney » Tue Jul 27, 2010 8:39 am
How do you quickly determine which values to pick and test?

User avatar
Legendary Member
Posts: 1893
Joined: Sun May 30, 2010 11:48 pm
Thanked: 215 times
Followed by:7 members

by kvcpk » Tue Jul 27, 2010 10:17 am
Sidney wrote:How do you quickly determine which values to pick and test?
Thats a good question.

I look at the statements and think critically, which combination can fail the condition.
I try choosing numbers normally in the following way:

Look at the boundaries of inequalities. Suppose 3<x<6, what is the reason that the writer of the question wants x to be confined between 3 and 6? there should be a problem if the numbers are chosen away from this range.

In normal problems without inequalities, I plugin in the following order:
Positive Integers
Zero
Negative Integers
Fractions (positive and negative)
Fractions between 0 and 1
Fractions between 0 and -1
Square roots

By the time you reach the end of this list, you will definitely have found the answer.

But the important thing is I plugin when I am almost sure that it is insufficient condition. I plugin to confirm that my analysis is right. A sufficient condition is difficult to prove with plugins. We will need to prove it mathematically.

Hope this helps!!

User avatar
Legendary Member
Posts: 748
Joined: Sun Jan 31, 2010 7:54 am
Thanked: 46 times
Followed by:3 members

by outreach » Tue Jul 27, 2010 12:17 pm
it has to be c or e since we need to know the values of x and y
x=1/5,1/9
y=-1/2,-1,0,1,1/2..1/5

x-y
1/5-(-1/2) >0
1/5-(1/5) = 0

hence E

sanju09 wrote:Is x - y > 0?
(1) 4 < 1/x < 10.
(2) -3 < 1/y < 6.


[spoiler]Source: Of course: writegmat.com[/spoiler]
-------------------------------------
--------------------------------------
General blog
https://amarnaik.wordpress.com
MBA blog
https://amarrnaik.blocked/

Master | Next Rank: 500 Posts
Posts: 131
Joined: Fri Jun 18, 2010 10:19 am
Location: New York, NY
Thanked: 10 times

by aleph777 » Tue Jul 27, 2010 1:41 pm
Interesting question.

We know that for x - y > 0 to be true, the difference must be 1 or greater.

Individually, both statements are insufficient. 1 only provides information about x; 2 only provides information about y.

Since the question only asks if x - y > 0, we don't care about narrowing in on specific numbers. But we can actually solve algebraically and easily by comparing the range of numbers in statements 1 and 2.

1: 4 < 1/x < 10
2: -3 < 1/y < 6

The two number ranges overlap, which means x could be greater than y... but y could be greater than x. And if y is greater than x, then x - y gives us a negative number. But since we can't be certain... The info together is insufficient.

ANSWER: E

Master | Next Rank: 500 Posts
Posts: 114
Joined: Mon Sep 22, 2008 3:51 am
Thanked: 8 times
GMAT Score:680

by Fiver » Wed Jul 28, 2010 10:35 am
Agree with the other posters.

Is x>y ?

Simple logical version- Is atleast the max 'y' greater than the min 'x'? If yes then the ans to our question is no
OR
Is the atleast the max 'x' greater than the min 'y'? If yes then the ans to our question is yes.

Quick glance suggests that neither stmts is suff.


St1] 4 < 1/x < 10.

St2] -3 < 1/y < 6.

Together we know that max 'y' is approaching 1/6 while min 'x' is approaching 1/10.
Also we know that max 'x' is approaching 1/4 while min 'y' is approaching -1/3.

Hence both the above scenarios are possible.

Choose E.